From: W. Trevor King Date: Tue, 4 Aug 2009 19:21:21 +0000 (-0400) Subject: Added problems for rec7 X-Git-Url: http://git.tremily.us/?a=commitdiff_plain;h=c2db38c1a2b379c6165029994831bc77b4498a71;p=course.git Added problems for rec7 --- diff --git a/latex/problems/Young_and_Freedman_12/problem26.61.tex b/latex/problems/Young_and_Freedman_12/problem26.61.tex new file mode 100644 index 0000000..39e7011 --- /dev/null +++ b/latex/problems/Young_and_Freedman_12/problem26.61.tex @@ -0,0 +1,17 @@ +\begin{problem*}{26.61} +Calculate the three currents $I_1$, $I_2$, and $I_3$ indicated in the +circuit diagram shown in Fig.~26.65. +\begin{center} +\begin{verbatim} + 5.00 8.00 ++------/\/\/------+------/\/\/------+ +|12.00V 1.00 I2 | I1 1.00 9.00V | ++--|i---/\/\/--<--+-->--/\/\/---i|--+ +| I3 10.00 | ++-------->------/\/\/---------------+ +\end{verbatim} +\end{center} +\end{problem*} + +\begin{solution} +\end{solution} diff --git a/latex/problems/Young_and_Freedman_12/problem26.86.tex b/latex/problems/Young_and_Freedman_12/problem26.86.tex new file mode 100644 index 0000000..3be73a8 --- /dev/null +++ b/latex/problems/Young_and_Freedman_12/problem26.86.tex @@ -0,0 +1,10 @@ +\begin{problem*}{26.86} +An $R$-$C$ circuit has a time constant $RC$. \Part{a} If the circuit +is discharging, how long will it take for its stored energy to be +reduced to $1/e$ of its initial value? \Part{b} If it is charging, +how long will it take for the stored energy to reach $1/e$ of its +maximum value? +\end{problem*} + +\begin{solution} +\end{solution} diff --git a/latex/problems/Young_and_Freedman_12/problem26.91.tex b/latex/problems/Young_and_Freedman_12/problem26.91.tex new file mode 100644 index 0000000..495eec2 --- /dev/null +++ b/latex/problems/Young_and_Freedman_12/problem26.91.tex @@ -0,0 +1,25 @@ +\begin{problem*}{26.91} +As shown in Fig.~26.83, a network of resistors of resistances $R_1$ +and $R_2$ extends to infinity toward the right. Prove that the total +resistance $R_T$ of the infinite network is equal to +\begin{equation} + R_T = R_1 + \sqrt{R_1^2 + 2R_1R_2} +\end{equation} +(\emph{Hint:} Since the network is infinite, the sestance of the +network to the right of points $c$ and $d$ is also equal to $R_T$.) +\begin{center} +\begin{verbatim} + R1 R1 R1 +a-/\/\/-c-/\/\/-+-/\/\/-+-... + | | | + Z Z Z + Z R2 Z R2 Z R2 + Z Z Z + R1 | R1 | R1 | +b-/\/\/-d-/\/\/-+-/\/\/-+-... +\end{verbatim} +\end{center} +\end{problem*} + +\begin{solution} +\end{solution} diff --git a/latex/problems/Young_and_Freedman_12/problem27.22.tex b/latex/problems/Young_and_Freedman_12/problem27.22.tex new file mode 100644 index 0000000..76916fd --- /dev/null +++ b/latex/problems/Young_and_Freedman_12/problem27.22.tex @@ -0,0 +1,13 @@ +\begin{problem*}{27.22} +In an experiment with cosmic rays, a verticle beam of particles that +have chagre of magnitude $3e$ and mass $12$ times the proton mass +enters a uniform horizontal magnetic field of $0.250\U{T}$ and is bent +in a semicircle of diameter $95.0\U{cm}$, as shown in +Fig.~27.47. \Part{a} Find the speed of the particles and the sign of +their charge. \Part{b} Is it reasonable to ignore the gravity force +on the particles? \Part{c} How does the speed of the particles as +they enter the field compare to their speed as they exit the field? +\end{problem*} + +\begin{solution} +\end{solution} diff --git a/latex/problems/Young_and_Freedman_12/problem27.30.tex b/latex/problems/Young_and_Freedman_12/problem27.30.tex new file mode 100644 index 0000000..e0d8e29 --- /dev/null +++ b/latex/problems/Young_and_Freedman_12/problem27.30.tex @@ -0,0 +1,12 @@ +\begin{problem*}{27.30} +A particle with initial velocity $\vect{v}_0=5.85\E{3}\U{m/s}\jhat$ +enters a region of uniform electric and magnetic fields. The magnetic +field in the region is $\vect{B}=-(1.35\U{T})\khat$. Calculate the +magnitude and direction of the electric field in the region if the +particle is to pass through undeflected, for a particle of +charge \Part{a} $+0.640\U{nC}$ and \Part{b} $-0.640\U{nC}$. You can +ignore the weight of the particle. +\end{problem*} + +\begin{solution} +\end{solution} diff --git a/latex/problems/Young_and_Freedman_12/problem27.35.tex b/latex/problems/Young_and_Freedman_12/problem27.35.tex new file mode 100644 index 0000000..bd1a6b1 --- /dev/null +++ b/latex/problems/Young_and_Freedman_12/problem27.35.tex @@ -0,0 +1,27 @@ +\begin{problem*}{27.35} +A long wire carrying $4.50\U{A}$ of current makes two $90\dg$ bends, +as shown in Fig.~27.49. The bent part of the wire passes through a +uniform $0.240\U{T}$ magnetic field direceted as shown in the figure +and confined to a limited region of space. Find the magnitude and +direction of the force that the magnetic field exerts on the wire. +\begin{center} +\begin{asy} +import ElectroMag; + +real u = 2.5cm; + +Distance Dhorizontal = Distance((0,0),(u,0), offset=2mm, L="$60.0\U{cm}$"); +Distance Dvertical = Distance((u,0),(u,u), offset=2mm, L="$60.0\U{cm}$"); +Distance Dbend = Distance((.25u,.25u),(.25u,.75u), offset=2mm, L="$30.0\U{cm}$"); + +draw(scale(u)*((0,0)--(1,0)--(1,1)--(0,1)--cycle), blue); +draw(scale(u)*((-.25,.25)--(.25,.25)--(.25,.75)--(1.25,.75)), red); +Dhorizontal.draw(labelangle=-90, labeloffset=8pt); +Dvertical.draw(labelangle=-90, labeloffset=8pt); +Dbend.draw(rotateLabel=false, labelangle=-90, labeloffset=22pt); +\end{asy} +\end{center} +\end{problem*} + +\begin{solution} +\end{solution}